K
Khách

Hãy nhập câu hỏi của bạn vào đây, nếu là tài khoản VIP, bạn sẽ được ưu tiên trả lời.

Ta có: 
1/(1+a)+1/(1+b)+1/(1+c)≥2 
→1/(1+a)≥{1-1/(1+b)}+{1-1/(1+c)} 
↔1/(1+a)≥b/(1+b)+c/(1+c) 
≥2.√(bc)/{(1+b)(1+c)}(theo cosi) 
Hai bất đẳng thức tương tự rồi nhân vế với vế 
1/{(1+a)(1+b)(1+c)≥8.abc/{(1+a)(1+b)(1... 
↔abc≤1/8(dpcm)

TK NHA

19 tháng 8 2017

\(\frac{1}{1+a}+\frac{1}{1+b}+\frac{1}{1+c}\ge2\Rightarrow\frac{1}{1+a}\ge\left(1-\frac{1}{1+b}\right)+\left(1-\frac{1}{1+c}\right)\)\(=\frac{b}{1+b}+\frac{c}{1+c}\ge2\sqrt{\frac{bc}{\left(1+b\right)\left(1+c\right)}}\)

Tương tự ta có: 

DD
22 tháng 1 2021

\(\frac{1}{a+1}\ge1-\frac{1}{b+1}+1-\frac{1}{c+1}=\frac{b}{b+1}+\frac{c}{c+1}\ge2\sqrt{\frac{bc}{\left(b+1\right)\left(c+1\right)}}\).

Tương tự ta có: \(\frac{1}{b+1}\ge2\sqrt{\frac{ac}{\left(a+1\right)\left(c+1\right)}}\)\(\frac{1}{c+1}\ge2\sqrt{\frac{ab}{\left(a+1\right)\left(b+1\right)}}\).

Nhân 3 bất đẳng thức trên theo vế ta được: 

\(\frac{1}{\left(a+1\right)\left(b+1\right)\left(c+1\right)}\ge\frac{8abc}{\left(a+1\right)\left(b+1\right)\left(c+1\right)}\)

\(\Leftrightarrow abc\le\frac{1}{8}\).

5 tháng 7 2020

Áp dụng bđt Cauchy cho 2 số không âm :

\(x^2+\frac{1}{x}\ge2\sqrt[2]{\frac{x^2}{x}}=2.\sqrt{x}\)

\(y^2+\frac{1}{y}\ge2\sqrt[2]{\frac{y^2}{y}}=2.\sqrt{y}\)

Cộng vế với vế ta được :

\(x^2+y^2+\frac{1}{x}+\frac{1}{y}\ge2.\sqrt{x}+2.\sqrt{y}=2\left(\sqrt{x}+\sqrt{y}\right)\)

Vậy ta có điều phải chứng mình 

5 tháng 7 2020

Ta đi chứng minh:\(a^3+b^3\ge ab\left(a+b\right)\)

\(\Leftrightarrow\left(a-b\right)^2\left(a+b\right)\ge0\)* đúng *

Khi đó:

\(\frac{1}{a^3+b^3+abc}\le\frac{1}{ab\left(a+b\right)+abc}=\frac{1}{ab\left(a+b+c\right)}=\frac{c}{abc\left(a+b+c\right)}\)

Tương tự:

\(\frac{1}{b^3+c^3+abc}\le\frac{a}{abc\left(a+b+c\right)};\frac{1}{c^3+a^3+abc}\le\frac{b}{abc\left(a+b+c\right)}\)

\(\Rightarrow LHS\le\frac{a+b+c}{abc\left(a+b+c\right)}=\frac{1}{abc}\)

4 tháng 10 2018

Ta có: \(\frac{1}{a+1}\ge2-\frac{1}{b+1}-\frac{1}{c+1}=\left(1-\frac{1}{b+1}\right)+\left(1-\frac{1}{c+1}\right)=\frac{b}{b+1}+\frac{c}{c+1}\ge2\sqrt{\frac{bc}{\left(b+1\right)\left(c+1\right)}}\)

Tương tự \(\frac{1}{b+1}\ge\frac{c}{c+1}+\frac{a}{a+1}\ge2\sqrt{\frac{ca}{\left(c+1\right)\left(a+1\right)}}\)

               \(\frac{1}{c+1}\ge\frac{a}{a+1}+\frac{b}{b+1}\ge2\sqrt{\frac{ab}{\left(a+1\right)\left(b+1\right)}}\)

Nhân từng vế, ta có: 

\(\frac{1}{\left(a+1\right)\left(b+1\right)\left(c+1\right)}\ge\frac{8abc}{\left(a+1\right)\left(b+1\right)\left(c+1\right)}\)

\(\Rightarrow abc\le\frac{1}{8}\)

13 tháng 11 2018

bé hơn hoặc bằng 1 hay là 2 vậy bạn

13 tháng 11 2018

\(\frac{a}{1+a}+\frac{b}{1+b}+\frac{c}{1+c}=3-\frac{1}{1+a}-\frac{1}{1+b}-\frac{1}{1+c}\le1\)

\(\Rightarrow T\frac{1}{1+a}\ge2\Rightarrow\frac{1}{1+a}\ge1-\frac{1}{1+b}+1-\frac{1}{1+c}=\frac{b}{1+b}+\frac{c}{1+c}\ge2\sqrt{\frac{bc}{\left(1+b\right)\left(1+c\right)}}\)

T là pháp cộng với b,c luôn nha, lười ghi.

Tương tự ta có:\(\frac{1}{1+b}\ge2\sqrt{\frac{ac}{\left(1+a\right)\left(1+c\right)}}\) và với c nữa

Nhân vế theo vế ta có đpcm

16 tháng 8 2018

Vô lí vì a+b+c=0\(\Rightarrow\frac{5}{a+b+c}\)không có đáp án

Y
19 tháng 5 2019

Theo BĐT AM-GM :

\(\sqrt{b}=\sqrt{b\cdot1}\le\frac{b+1}{2}\)

\(\Rightarrow\frac{a}{\sqrt{b}}\ge\frac{a}{\frac{b+1}{2}}=\frac{2a}{b+1}\)

Dấu "=" xảy ra \(\Leftrightarrow b=1\)

+ Tương tự ta cm đc :

\(\frac{b}{\sqrt{c}}\ge\frac{2b}{c+1}\). Dấu "=" xảy ra \(\Leftrightarrow c=1\)

\(\frac{c}{\sqrt{a}}\ge\frac{2c}{a+1}\). Dấu "=" xảy ra \(\Leftrightarrow a=1\)

Do đó : \(\frac{a}{\sqrt{b}}+\frac{b}{\sqrt{c}}+\frac{c}{\sqrt{a}}\ge2\left(\frac{a}{b+1}+\frac{b}{c+}+\frac{c}{a+1}\right)\)

Dấu "=" xảy ra \(\Leftrightarrow a=b=c=1\)

10 tháng 8 2018

Cho abc=0 thì không chứng minh được, a+b+c=0 là đủ rồi

Ta có: a+b+c=0 => a+b=-c

=>(a+b)2=(-c)2

=>a2+2ab+b2=c2

=>a2+b2-c2=-2ab

Tương tự ta có: b2+c2-a2=-2bc ; c2+a2-b2=-2ca

=>\(\frac{1}{b^2+c^2-a^2}+\frac{1}{c^2+a^2-b^2}+\frac{1}{a^2+b^2-c^2}=-\frac{1}{2bc}-\frac{1}{2ca}-\frac{1}{2ab}=\frac{a+b+c}{-2abc}=0\) (đpcm)

31 tháng 8 2018

Cho \(abc=0\)thì không chứng minh được, \(a+b+c=0\)là đủ rồi.

Ta có: \(a+b+c=0\Rightarrow a+b=-c\)

\(\Rightarrow\left(a+b\right)^2=\left(-c\right)^2\)

\(\Rightarrow a^2+2ab+b^2=c^2\)

\(\Rightarrow a^2+b^2-c^2=-2ab\)

Tương tự ta có: \(b^2+c^2-a^2=-2ab;c^2+a^2-b^2=-2ca\)

\(\Rightarrow\frac{1}{b^2+c^2-a^2}+\frac{1}{c^2+a^2-b^2}+\frac{1}{a^2+b^2-c^2}=-\frac{1}{2bc}-\frac{1}{2ca}-\frac{1}{2ab}=\frac{a+b+c}{-2abc}=0\)

5 tháng 2 2017

Áp dụng bất đẳng thức \(\frac{1}{a+b}\le\frac{1}{4}\left(\frac{1}{a}+\frac{1}{b}\right)\) với a , b > 0

\(\Rightarrow\left\{\begin{matrix}\frac{1}{a+b}\le\frac{1}{4}\left(\frac{1}{a}+\frac{1}{b}\right)\\\frac{1}{b+c}\le\frac{1}{4}\left(\frac{1}{b}+\frac{1}{c}\right)\\\frac{1}{a+c}\le\frac{1}{4}\left(\frac{1}{a}+\frac{1}{c}\right)\end{matrix}\right.\)

Cộng theo từng vế:

\(\Rightarrow\frac{1}{a+b}+\frac{1}{b+c}+\frac{1}{a+c}\le\frac{1}{4}\left(\frac{1}{a}+\frac{1}{a}+\frac{1}{b}+\frac{1}{b}+\frac{1}{c}+\frac{1}{c}\right)\)

\(\Rightarrow\frac{1}{a+b}+\frac{1}{b+c}+\frac{1}{c+a}\le\frac{1}{4}\left(\frac{2}{a}+\frac{2}{b}+\frac{2}{c}\right)\)

\(\Rightarrow\frac{1}{a+b}+\frac{1}{b+c}+\frac{1}{c+a}\le\frac{1}{2}\left(\frac{1}{a}+\frac{1}{b}+\frac{1}{c}\right)\) ( đpcm )

5 tháng 2 2017

Với a , b , c > 0

Ta có: \(a^2-2ab+b^2\ge0\)

\(\Rightarrow\) \(a^2+2ab+b^2\ge4ab\)

\(\Rightarrow\) \(\left(a+b\right)^2\ge4ab\)

\(\Rightarrow\) \(\frac{a+b}{4ab}\ge\frac{1}{a+b}\)

\(\Rightarrow\) \(\frac{1}{a+b}\le\frac{1}{4b}+\frac{1}{4a}\)

\(\Rightarrow\) \(\frac{1}{a+b}\le\frac{1}{4}\left(\frac{1}{a}+\frac{1}{b}\right)\)(1)

Chứng minh tương tự ta cũng có được:

\(\frac{1}{b+c}\le\frac{1}{4}\left(\frac{1}{b}+\frac{1}{c}\right)\) (2)

\(\frac{1}{a+c}\le\frac{1}{4}\left(\frac{1}{a}+\frac{1}{c}\right)\) (3)

Cộng (1), (2), (3) vế theo vế ta được:

\(\frac{1}{a+b}+\frac{1}{b+c}+\frac{1}{a+c}\le\frac{1}{4}\left(\frac{1}{a}+\frac{1}{b}+\frac{1}{b}+\frac{1}{c}+\frac{1}{a}+\frac{1}{c}\right)\)

\(\Rightarrow\) \(\frac{1}{a+b}+\frac{1}{b+c}+\frac{1}{a+c}\le\frac{1}{2}\left(\frac{1}{a}+\frac{1}{b}+\frac{1}{c}\right)\)( ĐPCM)